5
$\begingroup$

On my calculus test there was a problem asking to determine the convergence of $$\sum_{n=1}^{\infty} (-1)^n \cdot \frac{n!}{(2n - 1)!!}.$$

I looked on WolframAlpha after the test to see the sum surprisingly had a closed form, specifically $$\frac{1}{9} \left(-3 - 2\sqrt{3} \ \sinh^{-1} \left(\frac{1}{\sqrt{2}}\right) \right).$$

I would be interested in seeing how this is derived.

Note: As usual, for positive integers $k$, $k!!$ denotes the product of the integers from $1$ to $k$ that have the same parity as $k$ (in other words, the double factorial increments down by $2.$)

$\endgroup$
3
  • 4
    $\begingroup$A good starting point is to look up the taylor expansion of $\text{sinh}^{-1}$ and try to fix it to look like your formula, obtaining the weird corrections you notice.$\endgroup$Commentedyesterday
  • 1
    $\begingroup$Note that $$\frac{n!}{(2n-1)!!}=\frac{2^n}{\binom{2n}n}.$$ Not sure how that helps.$\endgroup$Commentedyesterday
  • $\begingroup$I have added a detailed Edit in my answer below to help OP. I also suggest OP to check his / her result at Wolfram Mathematica.$\endgroup$
    – Z Ahmed
    Commented9 hours ago

2 Answers 2

6
$\begingroup$

Following @Thomas Andrews and using $\frac{(n+1)}{{n\choose k}}=\int_{0}^{1}x^k (1-x)^{n-k}\,\mathrm dx,$ we get

$$S=\sum_{k=0}^\infty (-1)^k \frac{2^k}{{2k \choose k}}=\sum_{k=0}^{\infty}(2k+1)\int_0^{1}(-2x)^k (1-x)^{k}\,\mathrm dx.$$$$\implies S=\int_{0}^{1}\sum_{k=0}^{\infty}(2k+1) (2x^2-2x)^k\,\mathrm dx.$$ Next using $\sum_{k=0}^{\infty} z^k=(1-z)^{-1}, \sum_{k=0}^{\infty} k z^k= z(1-z)^{-2}, |z|<1$, we get $$\sum_{k=0}^{\infty} (2k+1) z^k=\frac{1+z}{(1-z)^2}, z=2(x^2-x).$$ Finally, $S$ is a doable definite integral, you may take over from here.

Edit:$$S=\int_{0}^{1} \frac{1+2x^2-2x}{(1-2x^2+2x)^2} dx$$ Let $x=\sin^2 t \implies dx=\sin 2t dt$, then $$S=2\int_{0}^{\pi/2} \frac{1+\cos^2 2t}{(3-\cos^2 2t)^2} \sin 2t~ dt.$$ Let $\cos 2 t=y$, then $$S=\int_{-1}^{1} \frac{1+y^2}{(y^2-3)^2} dy=2\int_{0}^{1} \left[ \frac{1}{y^2-3}+\frac{4}{(y^2-3)^2}\right] dy.$$ Use $\int \frac{dx}{x^2-a^2}=-\frac{1}{a} \tanh^{-1}(x/a)$ and $$\int \frac{dx}{(x^2-a^2)^2}=\frac{1}{2a^3}\left[\frac{ax}{a^2-x^2}+\tanh^{-1}(x/a)\right].$$$$S=\frac{2}{3}-\frac{2}{3\sqrt{3}} \tanh^{-1}\frac{1}{\sqrt{3}}$$$$S=\frac{2}{3}-\frac{2}{3\sqrt{3}} \sinh^{-1}\frac{1}{\sqrt{2}},$$ which matches with what the Wolfram Mathematica would give. I suggest OP to check his expression obtained at Mathematica.

$\endgroup$
2
  • 1
    $\begingroup$Thank you for writing out all of the details. I believe the slight differences in the answers is the difference in starting indices.$\endgroup$Commented8 hours ago
  • $\begingroup$Oh, yes you are write the difference is due to starting $k$ from 1 (yours) or from 0 (mine). Cheers$\endgroup$
    – Z Ahmed
    Commented2 mins ago
0
$\begingroup$

$$\frac{n!}{(2 n-1)\text{!!}}=\frac{\sqrt{\pi } \, \Gamma (n+1)}{2^n\,\Gamma\left(n+\frac{1}{2}\right)}$$ Consider $$\sum_{n=0}^\infty \frac{\sqrt{\pi } \, \Gamma (n+1)}{\Gamma\left(n+\frac{1}{2}\right)}\,x^{2n}$$ Cheating a little $$\sum_{n=0}^\infty \frac{\sqrt{\pi } \, \Gamma (n+1)}{\Gamma\left(n+\frac{1}{2}\right)}\,x^{2n}=\frac{\sqrt{1-x^2}+x \sin^{-1}(x)}{\left(1-x^2\right)^{3/2}}$$

$$x=\frac{i}{\sqrt{2}}\quad \implies \quad \sum_{n=0}^\infty (-1)^n\frac{n!}{(2 n-1)\text{!!}}=\frac{2}{3}-\frac{2 \sinh ^{-1}\left(\frac{1}{\sqrt{2}}\right)}{3 \sqrt{3}}$$ Subtract the first term which is $1$ to get the result.

$\endgroup$

    You must log in to answer this question.

    Start asking to get answers

    Find the answer to your question by asking.

    Ask question

    Explore related questions

    See similar questions with these tags.